This is a most strongly supported question, as the question stem asks: If both Monroe’s conclusion and the evidence on which he bases it are correct, they would provide the strongest support for which one of the following?

From the question stem alone we already know someone named Monroe is going to be giving us a conclusion supported by some evidence. The first sentence gives us a phenomenon; Monroe ate at the Tip-Top Restaurant and enjoyed the meals, but afterwards became ill each time. He must have really enjoyed it to go back after being sick two times in a row! Since we have started with a phenomenon, Monroe getting sick every time he eats at Tip-Top, we should expect a causal hypothesis as to why this correlation is occurring. The next sentence gives us some more detail about this correlation. Monroe ate three different meals at Tip-Top, but each one had hot peppers. So this Monroe getting sick every time he eats at Tip-Top correlation is also more specifically a Monroe getting sick every time he eats hot peppers correlation. Where the first sentence probably made you think, “Maybe you should try a new restaurant, Monroe!”, this new information has properly altered your hypothesis to, “Maybe ask them to skip the hot peppers next time, Monroe!”. The final sentence tells us that Monroe agrees and gives us his conclusion, his hypothesis for why he kept getting sick; it was the hot peppers causing him to feel sick after all three meals. He probably should have been able to figure that out after two meals, but good for Monroe!

Having now read the stimulus, and remembering that we are supposed to assume Monroe’s hot pepper hypothesis is correct, our job is to select the answer that is most strongly supported based off of it. Let’s take a look at the answer choices:

Answer Choice (A) What this answer choice does is confuse sufficiency with necessity. Just because hot peppers are enough for Monroe to feel sick, doesn’t mean they are required for him to be sick. Maybe he’s also allergic to pine nuts and the all-you-can-eat specials includes a really nice Pesto pasta! If you chose this answer, you might want to review conditional logic.

Correct Answer Choice (B) This is the correct answer because it does what A tries to do without its error of confusing enough with required. Where A said Monroe can eat anything without hot peppers and not get sick, B correctly infers that if Monroe ate a different dish with hot peppers he would have still gotten sick, because hot peppers are enough for him to be ill.

Answer Choice (C) Same issue as A, this question incorrectly assumes that hot peppers are the only possible cause of Monroe feeling ill.

Answer Choice (D) We need to assume that we just don’t know about Monroe’s eating history for this answer to follow from our stimulus.

Answer Choice (E) This should be clearly wrong. We know nothing about Monroe’s restaurant history outside of Tip-Top. Maybe Monroe is really bad at noticing when an ingredient makes him feel sick and he’s actually eaten hot peppers 20 times at other restaurants and gotten sick, and only now after three meals at Tip-Top made the connection.


1 comment

This is a most strongly supported question, since the question stem says: Which one of the following conclusions is most strongly supported by the results of the experiments?

As you should have noticed in the question stem, this stimulus involves experiments. We should therefore be on the lookout for a phenomenon. The first sentence begins with the conditional indicator only, and from it we learn that some strains of tobacco are resistant to tobacco mosaic virus; important to note that resistant here doesn’t mean they resist infection, but rather they resist developing symptoms and becoming diseased. This resistance is our phenomenon. The first of the experiments gives us a correlation. Tobacco plants infected with the mosaic virus who resisted its effects had increased levels of salicylic acid, while non-resistant plants had no increase in their levels. Resistance to the virus correlates with more salicylic acid. The second experiment greatly strengthens the correlation by artificially raising the salicylic levels of half of 50 non-resistant plant, with every high salicylic level plant resisting the disease and every normal plan succumbing to it.

If we were to summarize this stimulus, it is essentially: phenomenon, correlation, even stronger correlation. What we never receive in this stimulus is a conclusion, or more specifically in this case, a hypothesis. If we pre-phrase, we would expect a conclusion to this information to infer causation from correlation, and conclude that the increased salicylic acid at least partly causes the mosaic resistance. We should be on the lookout for an answer that introduces this hypothesis, while eliminating any answers that have little to no support in the stimulus. Let’s take a look at our options:

Answer Choice (A) None of the experiments involved salicylic acid removing symptoms, but only preventing them. Our correlation is between salicylic acid and resistance, not healing.

Correct Answer Choice (B) This answer does exactly what we should have predicted in our pre-phrasing; it introduces the expected hypothesis that salicylic acid at least partly explains why some plants are able to resist the mosaic virus.

Answer Choice (C) This information is not only not supported, but likely false based on the stimulus. The salicylic acid is described as naturally occurring in the plants, and the difference between the resistant and non-resistant plants is whether there was an increase in salicylic acid.

Answer Choice (D) Remember we are told about an increase in the salicylic levels following infection; if the plant is uninfected the stimulus suggests that its salicylic acid levels will be normal, and maybe even indistinguishable from those of non-resistant plants.

Answer Choice (E) This answer might be appealing since it does emphasize the correlation between an increase in salicylic acid and resistance to the virus. The problem is that the experiments involved artificially injecting plants to raise their salicylic levels, and we have no information about whether it is possible increase the plant’s production of the acid.


Comment on this

This page shows a recording of a live class. We're working hard to create our standard, concise explanation videos for the questions in this PrepTest. Thank you for your patience!

This is a weakening question, since we are asked: Which one of the following, if true, most seriously weakens the argument?

This stimulus is relatively straightforward. We are first told that a portion of economically useful raw materials are both nonrenewable and in limited supply on Earth. So we can’t make more of these resources and we don’t have such a big supply that we would never run out. Following this premise we get a sentence beginning with the conclusion indicator therefore and the conditional indicator unless. If we translate this conditional conclusion, we should end up with something along the lines of: no materials can be obtained outside earth → unable to accomplish what is now accomplished with those materials (referential phrase: the useful, non-renewable, and limited supply materials).

When we are faced with a weakening question involving a standard argument, we should always consider if the argument relies on a large assumption. In this case, we are concluding that if we can’t find more of these materials, we won’t be able to do what we use them for right now. The major assumption here is that these materials are required for what we do with them. For example, just because we currently use gas to produce electricity doesn’t mean that gas is required to produce electricity. An easy way to weaken this argument is to explicitly state that these materials aren’t required, and could be replaced with alternatives. We should be on the lookout for an answer choice that does this. Let’s see what we get:

Answer Choice (A) The argument we are trying to weaken is about the portion of economically useful materials which aren’t renewable. Even if there are renewable resources, unless they are viable alternatives to our non-renewables, we don’t care about them.

Answer Choice (B) This answer choice just suggests that the conditional conclusion of our argument will be triggered, since it is hard to get resources outside of earth. This does nothing to weaken or even strengthen the argument.

Correct Answer Choice (C) This contradicts the assumption we identified in the stimulus. If these limited nonrenewable resources can be replaced with alternatives, then even if we can’t find more of them we can still keep doing what we are doing with them. In short, this says the resources aren’t required, which is exactly what the argument assumes they are.

Answer Choice (D) Who cares? The argument we are trying to weaken has a conditional conclusion; it makes a prediction about what will happen in a particular case. The worthiness of what the resources are being used for is irrelevant to whether it will be impossible to continue should they run out.

Answer Choice (E) This answer fills in one of the assumptions the argument makes; that we will ever run out of these resources just because there is a limited supply of them. For that reason, if anything, all this answer does is strengthen what we want to weaken.


Comment on this

This page shows a recording of a live class. We're working hard to create our standard, concise explanation videos for the questions in this PrepTest. Thank you for your patience!

We can identify this question as Method of Reasoning because of the question stem: “In countering the original conclusion the reasoning above uses which one of the following techniques?”

When dealing with a Method of Reasoning question, we know we are looking for an answer choice that correctly describes the structure of our entire argument. Our correct answer is going to fit the argument exactly. Our wrong answer choices likely explain argument structures we are familiar with, but that simply don’t apply to the specific question we are looking at. Knowing what the right and wrong answers are going to do, we can jump into the stimulus.

The argument begins by telling us about a correlation; those who play bridge tend to have better short-term memories than those who do not play bridge. We know right away that the presence of two qualities at the same time does not mean they caused each other. The argument proceeds by affirming this exactly. We are told that although this correlation was previously concluded to indicate a causal relationship meaning bridge causes better short-term memory, it is just as likely that having a better short-term memory makes the game of bridge more intriguing to that particular group of people.

Ultimately, our stimulus outlines an incorrect conclusion on the basis of a correlation and then goes on to explain another possible interpretation from the evidence. Knowing this, we can jump into answer choice elimination.

Answer Choice (A) If our stimulus were challenging the representativeness of the study included, we would expect the argument to bring up the number of people involved in these different groups or how well these groups represent the rest of the population. Without this information we can eliminate answer choice A.

Answer Choice (B) This answer accuses our stimulus of drawing a conclusion about what is considered “appropriate therapy.” Without this emphasis in the text we can eliminate B.

Answer Choice (C) Our stimulus does not depend on some sort of misunderstanding of the facts involved in the scenario. Thus, we can nix answer choice C.

Correct Answer Choice (D) This is exactly what we are looking for! This is the only answer choice that identifies the alternative possible explanations for the facts presented in the argument.

Answer Choice (E) This answer choice is tricky. But our stimulus does not go quite far enough to say it is describing a flaw of the previous conclusion. Because our stimulus only points out the existence of another possible conclusion we cannot confirm answer choice E.


Comment on this

This page shows a recording of a live class. We're working hard to create our standard, concise explanation videos for the questions in this PrepTest. Thank you for your patience!

We can identify this question as Method of Reasoning because of the question stem: “Jonathan uses which one of the following techniques in his response to Lydia?”

When dealing with a Method of Reasoning question, we know we are looking for an answer choice that correctly describes the structure of our entire argument. Our correct answer is going to fit the argument exactly. Our wrong answer choices likely explain argument structures we are familiar with, but that simply don’t apply to the specific question we are looking at. Knowing what the right and wrong answers are going to do, we can jump into the stimulus.

Immediately we should make note of the two speakers at play. This means we could possibly be dealing with two different conclusions with different levels of support. Our first speaker, Lydia, tells us seabirds often become entangled in equipment owned by fishing companies. Lydia concludes on the basis of this that the fishing companies should assume responsibility for the medical treatment of these animals.

Lydia’s position makes an assumption here. If our conclusion tells us that something should happen, our evidence needs to give us reasoning to guarantee the outcome should occur. Perhaps, for instance, there is a law indicating those causing harm to animals should be responsible for them. But without this information the evidence does not automatically lead to the conclusion that the fisherman should be responsible for anything.

Jonathan does not quite hit the assumption out of the ballpark. In response, our second speaker concludes the proposal should not be adopted because the most injured birds won’t be able to return to the wild. Remind yourself here of how uncertain the number of “most injured birds” is. Perhaps 99.99% of the birds are injured mildly and 0.01% are the “most injured” with extensive injuries. Putting things into context, Jonathon’s response asking us to consider a group that could be impossibly small and irrelevant to Lydia’s ultimate conclusion.

Knowing that we are looking for an answer choice that will highlight Jonathon’s use of a small subset of these animals in a (poor) attempt to weaken Lydia’s reasoning we can jump into answer choice elimination.

Answer Choice (A) This answer choice accuses Lydia of a personal attack. But without any reference to Lydia’s motivation or other personal characteristics, we have to eliminate this answer from contention.

Answer Choice (B) We can eliminate this answer for a similar reason why we eliminated answer choice A. Like a personal attack, B accuses Lydia of being wrapped up in their personal interests - an attack we do not see used as the reasoning for Jonathon’s conclusion.

Answer Choice (C) This answer choice goes too far in the extreme. By accusing our second speaker of not wanting to interfere with wildlife in any way, this answer choice claims Jonathon’s conclusion goes even further than we can see in the stimulus.

Correct Answer Choice (D) This is exactly the answer we are looking for! This is the only answer choice that references Jonathon’s use of the sickest group of birds in an attempt to weaken Lydia’s argument.

Answer Choice (E) While Lydia’s feelings are addressed at the beginning of Jonathan’s argument, this is not the reasoning used to prove Jonathon’s main point. They claim we should not adopt the proposal because of this sub-group of birds. Not because of Lydia’s personal feelings in the matter.


1 comment

This page shows a recording of a live class. We're working hard to create our standard, concise explanation videos for the questions in this PrepTest. Thank you for your patience!

We can identify this question as Method of Reasoning because of the question stem: “The argument uses which one of the following argumentative techniques?”

When dealing with a Method of Reasoning question, we know we are looking for an answer choice that correctly describes the structure of our entire argument. Our correct answer is going to fit the argument exactly. Our wrong answer choices likely explain argument structures we are familiar with, but that simply don’t apply to the specific question we are looking at. Knowing what the right and wrong answers are going to do, we can jump into the stimulus.

The philosopher begins their argument by explaining a belief of the thesis absolute motion; the change in an object’s position over time does not need a reference item for measurement. We are told a well-respected physicist claims the thesis to be incoherent. The philosopher concludes that because an incoherent thesis cannot be a description of reality, the thesis is incorrect (motion cannot be absolute.) Knowing that our stimulus uses the belief of the physicist to support their argument, we can proceed into answer choice elimination.

Answer Choice (A) This answer choice is not descriptively accurate. Without the use of “technological terminology” in the stimulus, we can eliminate answer choice A.

Answer Choice (B) Our stimulus is not suggesting a change in definition as suggested by this answer choice. Instead the argument tells us that a particular thesis is not correct.

Correct Answer Choice (C) This is exactly what we are looking for! This is the only answer choice that correctly describes the author’s use of an authority (the physicist) to support the main conclusion.

Answer Choice (D) If this answer choice were correct, we would need to see the use of some sort of observational evidence in the stimulus. Without this information, we can eliminate answer choice D from contention.

Answer Choice (E) This is not descriptively accurate. This answer choice accuses the stimulus of referring to different regions and comparing those to the entire region. Without any sort of “region” distinction we will have to eliminate answer choice E.


Comment on this

This page shows a recording of a live class. We're working hard to create our standard, concise explanation videos for the questions in this PrepTest. Thank you for your patience!

We can identify this question as Method of Reasoning because of the question stem: “Myrna responds to Roland by…”

When dealing with a Method of Reasoning question, we know we are looking for an answer choice that correctly describes the structure of our entire argument. Our correct answer is going to fit the argument exactly. Our wrong answer choices likely explain argument structures we are familiar with, but that simply don’t apply to the specific question we are looking at. Knowing what the right and wrong answers are going to do, we can jump into the stimulus.

Immediately we should make note of the two speakers at play. This means we could possibly be dealing with two different conclusions with different levels of support. Our first speaker, Myrna, begins the discussion by introducing a claim; people’s calorie breakdown should consist of 30% or less of fat compared to the 37% that makes up a person’s diet in this country on average.

Roland responds to this claim with a hypothetical. The second speaker explains if everyone were to follow this recommendation, a very small percent of people would live a tiny bit longer than they would otherwise. Ronald concludes that such a sacrifice is not worthwhile as a result. Here, Ronald makes an assumption. By telling us that a possible 3 month extension is not worthwhile, Roland has assumed there are no possible worthwhile benefits aside from living for another 3 months.

Myrna points out this assumption in their reply. The speaker tells us a low-fat diet not only has the potential to extend life expectancy, but also it has the ability to reduce the recurrence of diseases that are impacting people on a daily basis. Myrna weakens Roland’s position by pointing out there are other benefits that may not be directly related to life expectancy. Having broken down the arguments of our speakers, we can proceed into answer choice elimination.

Answer Choice (A) If Mryna were disputing the correctness of the facts we would expect them to introduce some information that misaligned with Roland’s argument. Because of this we can eliminate answer choice A.

Correct Answer Choice (B) This is exactly what we are looking for! This is the only answer choice that highlights Myrna’s method to have Roland consider benefits outside of life expectancy.

Answer Choice (C) Similar to answer choice A, this answer accuses Myrna of disputing the factual correctness of Roland’s statistics. Because Myrna introduces a completely new topic of consideration we can eliminate this answer choice.

Answer Choice (D) We do not see Mryna refer to the sources used to create Roland’s argument. We can eliminate answer choice D because of this.

Answer Choice (E) This answer choice accuses Myrna of engaging in circular reasoning - where we use the conclusion as the evidence for our conclusion. Without some sort of circular train of thought (we should follow the diet because we should follow the diet) we can eliminate this answer choice from contention.


Comment on this

This page shows a recording of a live class. We're working hard to create our standard, concise explanation videos for the questions in this PrepTest. Thank you for your patience!

This is a Sufficient Assumption (SA) question and we know this because of the question stem: “… an assumption that would make the conclusion in the passage a logical one?”

Sufficient assumption questions tend to be very formal. We’re looking for a rule that would validate (not just strengthen) the conclusion, specifically by bridging the premise and conclusion through the rule. Not only are we extrapolating the rule from our argument, but we’re also using that rule to render the argument “valid.” The way to prephrase our answer choice is by tying our premises and conclusion together into a rule: “If [premise] à then [conclusion].”

Our first sentence is describing how some accountants use adding machines while some use computers. I can’t give examples of the two, but it doesn’t matter. If it did, the stimulus would specify. What is important about the difference between the two is given in the next sentence: complex computers are faster than adding machines: they’re efficient compared to adding machines as we can do more in less time. So far, this makes sense.

The next sentence begins with a conclusion indicator. I haven’t seen anything that looks like a conclusion, but let’s read the whole sentence before we put labels on anything. The author explicitly assumes that the costs of the two machines are equal and then claims that accountants who use complex computers earn more per hour than accountants who use adding machines.

Hang on a minute – there’s a huge gap here between doing more calculations per hour and earning more per hour. These two things aren’t necessarily related. What if they work on salary and their bonuses do not depend on how much work they do? What if they only get a fixed number of clients and finishing work sooner rather than later has no bearing on how much they will earn?

If order to bridge this gap in our prephrase, we can simply say something like “the more calculations accountants are able to do, the more they can earn.

Answer Choice (A) This is incorrect. Our rule doesn’t care about the number of accountants that are using the complex machine. The rule wants to address the gap between using the machine and how it affects earnings.

Answer Choice (B) This is close, but it’s wrong. Earnings are affected by the number of calculations one can perform, it’s not about the number of hours. If it was about the number of hours, it would be better to use a slower calculator as it would take more time to do the job.

Correct Answer Choice (C) It’s a paraphrase of our prephrase.

Answer Choice (D) This is not correct because this would weaken the conclusion: they’d be able to charge more using the adding machine and, assuming the price charged is proportional to earnings, they would be able to earn more.

Answer Choice (E) This answer choice establishes a vague relationship between earning and money. It’s not enough. We need a positive relationship between the two.


Comment on this

This page shows a recording of a live class. We're working hard to create our standard, concise explanation videos for the questions in this PrepTest. Thank you for your patience!

This is a sufficient assumption question because the question stem says: “conclusion follows logically…if which one… is assumed?”

Sufficient assumption questions tend to be very formal. We’re looking for a rule that would 100% validate the conclusion, specifically by bridging the premise and conclusion through the rule. Not only are we extrapolating the rule from our argument, but we’re also using that rule to render the argument “valid.” The way to prephrase our answer choice is by tying our premises and conclusion together into a rule: “If [premise] → then [conclusion].”

Our first sentence is very straightforward: in bureaucracies, decisions involve many people. The sentence just elaborates on this by saying that no one person have more authority than the next person. These are both our premises.

Our conclusion states that in bureaucracies, risky projects are never undertaken. That’s a big jump! From “bureaucracy decisions involving multiple people” to concluding “risky projects are never undertaken”? What if that is the exact they take on risky projects? For example, they could say that because many people are involved, they’re able to prepare for every outcome.

In order for our conclusion to follow, we need to link up the idea in the premises to the conclusion: when multiple people are involved in decisions and no one has the authority, risky projects will not be taken.

Answer Choice (A) This isn’t correct. We’re trying to make our conclusion about risky projects not being taken on valid; the fact that projects always require risk doesn’t help the gap in our argument.

Answer Choice (B) This isn’t correct either. The gap here is that the argument assumes that have many people involved in a decision means that no risky project will be taken. If we plug this into our stimulus, this doesn’t help validate our conclusion. It could support it, but there are too many assumption we need to make to arrive to our conclusion.

Answer Choice (C) This is more or less what our premises are trying to say, but again, it’s not helping to validate our conclusion about risky project not being taken on in bureaucracies. Additionally, we’re not concerned with what groups of people will take risks - we specifically interested in bureaucracies who will not take risks.

Correct Answer Choice (D) We said we weren’t interested in people who take risks, but this is a conditional statement! The answer choice is saying “when risk take, then single individual power to decide.” Taking the contrapositive of this would be: if multiple people have the power to decide, the risk is not taken. See how “no risk taken” is in the necessary condition? This is the NC in our rule, and it’s also our conclusion. Our premises trigger the contrapositive of this answer and allow us to draw our conclusion.

Answer Choice (E) This isn’t correct; what people do on there own is outside the scope of their decision as a group. This doesn’t help us draw our conclusion.


Comment on this

This is a necessary assumption question because the stem tells us that the argument “depends on the assumption.”

Necessary assumption questions fall under the subset of strengthen questions. All of the things we learned for strengthen, SA, and PSA question are still very important here! We’re going to be analyzing the stimulus the same way: identify the premise and conclusion, evaluate the argument, and determine what is missing. Our approach to the questions is very different. For NA question, in order for our conclusion to be true, our correct answer must be true. Without the correct answer, our argument will fall apart. This is what we’re looking for in our answer choice. Remember, we can always test our answer choices by using the negation test: if we negate the answer choice and it destroys our argument, it is the correct answer!

The first sentence is pretty straight-forward: the permits are issued in terms of pounds of each chemicals that can be dumped into the waterway per day. So, each chemical has a specified weight that can be dumped in the water each day. We’re also told how these numbers are calculated: by looking at the effects dilution in the water based on the amount of water flowing through the waterway. So there’s two variables here: the dilution and how much water passes through the waterway. This makes sense: if you have more/less water flowing through the waterway, dilution will change. Great! The next sentence is the conclusion: based on all of this information, the waterway is protected.

On the LSAT, there is a correct amount of skepticism you need to have. The argument is okay but, while it’s difficult to pinpoint, there is a gap here. A number of things could happen in order for this conclusion to not be true. For example, what if no one follows these guidelines? What if calculation of water flowing through the waterway isn’t accurate because the flow of water changes through the day? Any number of things need to be true if our conclusion, the waterway is protected, is true.

Correct Answer Choice (A) This is necessary for the argument! If, even in safe quantities, chemicals interact to form harmful ones, that would destroy our argument.

Answer Choice (B) Is rapid dispersion necessary? Maybe the flow is very slow, but the amount of chemicals allowed in the waterway is very low. This is not good.

Answer Choice (C) This is also not necessary. Let’s take the negation of this sentence: some chemicals are prohibited from being discharged into the waterway. So, what? Perhaps arsenic isn’t not allowed to be dumped - is that compatible with the argument? Yes! Prohibiting certain chemicals to be dumped in the waterway does not destroy the argument.

Answer Choice (D) This is also not necessary. The permits indicate the max amount that can be dumped into the water, which means dumping anything at the maximum amount or below is fine. Negating this sentence (they dump the full amount indicated by the permit) does not destroy the argument.

Answer Choice (E) This weakens our argument a little bit - it mentions what we’re not taking into consideration in an effort to say that the waterway may not be as protected as the argument says. This is out.

 


Comment on this